2
$\begingroup$

Let $\Gamma$ be a discrete group.

Q: If $l^\infty(\Gamma)\rtimes \Gamma=l^\infty(\Gamma)\rtimes_r \Gamma$ canonically, can we conclude that $\Gamma$ is an exact group?

The converse implication is well-known and if we replace $l^\infty(\Gamma)$ by $l^\infty(\Gamma)/C_0(\Gamma)$ we have indeed an affirmative answer to the question. The latter fact can be found in Section 5 of "Ghostbusting and property A" by Roe and Willett or Theorem 5.6 in http://arxiv.org/pdf/1603.01829.pdf

$\endgroup$
6
  • $\begingroup$ By "canonically isomorphic", do you mean that "the canonical map from the full to the reduced cross product is an isomorphism"? $\endgroup$
    – Yemon Choi
    Mar 12, 2016 at 0:56
  • $\begingroup$ @m07kl Yes. See Theorem 4.4.3 in Brown and Ozawa's book, C*-algebras and finite dimensional approximation properties $\endgroup$ Mar 12, 2016 at 4:53
  • $\begingroup$ @YemonChoi Yes. :) $\endgroup$
    – m07kl
    Mar 12, 2016 at 12:14
  • $\begingroup$ @CalebEckhardt: I don't think you can apply Theorem 4.4.3, because here you only use one $C^*$-algebra namely $l^\infty(\Gamma)$. Please see arxiv.org/abs/1204.3050 for a partial solution. $\endgroup$
    – m07kl
    Mar 12, 2016 at 12:21
  • 1
    $\begingroup$ The answer is affirmative and it follows from a very recent paper by Anantharaman-Delaroche see arXiv:1604.01724 $\endgroup$
    – m07kl
    Apr 21, 2016 at 18:08

1 Answer 1

0
$\begingroup$

See final remark of "SOME REMARKS ABOUT THE WEAK CONTAINMENT PROPERTY FOR GROUPOIDS AND SEMIGROUPS" by Claire Anantharaman-Delaroche for an affirmative answer of the question.

https://arxiv.org/pdf/1604.01724.pdf

edit (by AlexE): the cited argument is flawed, see my comment below.

$\endgroup$
2
  • $\begingroup$ It looks like a nice argument. Why not give an outline of the argument here? $\endgroup$
    – Yemon Choi
    May 18, 2016 at 16:34
  • 1
    $\begingroup$ The final remark that you refer to is flawed. In fact, in the new version of the paper you don't find it anymore. Concretely, I was told by Kang Li that Cor. 2.12 in version 3 of the paper is wrong, which invalidates the final step in the final remark you mention. As far as I know, your question is currently still open. $\endgroup$
    – AlexE
    Nov 19, 2020 at 11:11

Your Answer

By clicking “Post Your Answer”, you agree to our terms of service and acknowledge you have read our privacy policy.

Not the answer you're looking for? Browse other questions tagged or ask your own question.